The author of the passage would be most likely to agree with which one of the following statements regarding the subs...

tomgbean on December 18, 2019

A E and D

So the first time around I picked A but I understand that it's wrong because the passage does not suggest the modular theory of mind could not be used for medical application. Then I chose E because in the last paragraph the author states that one problem with the subtractive method is that it masks the fact that the larger distributed portions of the brain are active in both the controlled state and during the time of activities and that the subtractive method masks this. Is E wrong because of the phrase "fundamental misconception of the method?" I took that to mean that the method is improperly conceived or that the method is flawed because it does not reveal the whole truth about brain activities. I think I understand why D is correct. It is directly stated in the passage...but could you rephrase why A and E are incorrect in your own words so I can see reasoning that may be different than my own?

Reply
Create a free account to read and take part in forum discussions.

Already have an account? log in

shunhe on December 23, 2019

Hi @tomgbean,


The problem with (E) is because it is not a fundamental misconception of the method that it depicts differential rate of oxygen use - actually, that is the right way to conceive it! If we take a look at lines 35-40, we see that the actual misconception is to see it as depicting rates of oxygen use. So this is why (E) is wrong. (A), on the other hand, is far too strong of a statement. Nowhere does the author argue that empirical results of the subtractive method are invalid for medical applications. Hope this helps!